Ultraproducts with a non-principal ultrafilter is aleph-one compact












2












$begingroup$


I came across the following statement:




Given an $omega$-sequence of $mathcal{L}$-structures, $(mathcal{M}_i : i < omega)$, and a non-principal ultrafilter $mathcal{U}$ on $omega$, the ultraproduct of the $mathcal{L}$-structures,



$$ mathcal{M} = prod_{mathcal{U}} M_i $$
$aleph_1$-compact, i.e. for any countable (non-empty) collection of non-empty definable sets in $mathcal{M}$ that has the Finite Intersection Property, the collection has a non-empty intersection.




Perhaps the given statement was incomplete (it was in the middle of a lecture, as an aside), but in its proof, it is assumed that the collection is nested, and the rest of the proof inherently relies on this property.



I wonder if the "nestedness" of the collection is actually a necessary condition. I set out for a proof, but I am unable to arrive at a conclusion. Here is an outline of my attempt:



Let $(F_n)_{n < omega}$ be the collection given by the assumption, and let $phi_n$ be the $mathcal{L}$-formula that defines $F_n$, for each $n < omega$. Then for any $N < omega$, by the Finite Intersection Property, we can find a realization $[a_N] in bigcap_{n = 0}^{N} F_n$ such that



$$mathcal{M} models bigwedge_{n=0}^{N} phi_n ([a_N]).$$



By Łoś, we have



$$ left{ i in I : mathcal{M}_i models bigwedge_{n = 0}^{N} phi_n(a_N(i)) right} in mathcal{U}. $$



That is where I am stuck at. I reckon that my goal is to perhaps show that



$$left{ i in I : mathcal{M}_i models bigwedge_{n < omega} phi_n(a'(i)) right} in mathcal{U}$$



and complete the proof by Łoś, but I am not entirely certain if that is what I should be pursuing.










share|cite|improve this question









$endgroup$

















    2












    $begingroup$


    I came across the following statement:




    Given an $omega$-sequence of $mathcal{L}$-structures, $(mathcal{M}_i : i < omega)$, and a non-principal ultrafilter $mathcal{U}$ on $omega$, the ultraproduct of the $mathcal{L}$-structures,



    $$ mathcal{M} = prod_{mathcal{U}} M_i $$
    $aleph_1$-compact, i.e. for any countable (non-empty) collection of non-empty definable sets in $mathcal{M}$ that has the Finite Intersection Property, the collection has a non-empty intersection.




    Perhaps the given statement was incomplete (it was in the middle of a lecture, as an aside), but in its proof, it is assumed that the collection is nested, and the rest of the proof inherently relies on this property.



    I wonder if the "nestedness" of the collection is actually a necessary condition. I set out for a proof, but I am unable to arrive at a conclusion. Here is an outline of my attempt:



    Let $(F_n)_{n < omega}$ be the collection given by the assumption, and let $phi_n$ be the $mathcal{L}$-formula that defines $F_n$, for each $n < omega$. Then for any $N < omega$, by the Finite Intersection Property, we can find a realization $[a_N] in bigcap_{n = 0}^{N} F_n$ such that



    $$mathcal{M} models bigwedge_{n=0}^{N} phi_n ([a_N]).$$



    By Łoś, we have



    $$ left{ i in I : mathcal{M}_i models bigwedge_{n = 0}^{N} phi_n(a_N(i)) right} in mathcal{U}. $$



    That is where I am stuck at. I reckon that my goal is to perhaps show that



    $$left{ i in I : mathcal{M}_i models bigwedge_{n < omega} phi_n(a'(i)) right} in mathcal{U}$$



    and complete the proof by Łoś, but I am not entirely certain if that is what I should be pursuing.










    share|cite|improve this question









    $endgroup$















      2












      2








      2





      $begingroup$


      I came across the following statement:




      Given an $omega$-sequence of $mathcal{L}$-structures, $(mathcal{M}_i : i < omega)$, and a non-principal ultrafilter $mathcal{U}$ on $omega$, the ultraproduct of the $mathcal{L}$-structures,



      $$ mathcal{M} = prod_{mathcal{U}} M_i $$
      $aleph_1$-compact, i.e. for any countable (non-empty) collection of non-empty definable sets in $mathcal{M}$ that has the Finite Intersection Property, the collection has a non-empty intersection.




      Perhaps the given statement was incomplete (it was in the middle of a lecture, as an aside), but in its proof, it is assumed that the collection is nested, and the rest of the proof inherently relies on this property.



      I wonder if the "nestedness" of the collection is actually a necessary condition. I set out for a proof, but I am unable to arrive at a conclusion. Here is an outline of my attempt:



      Let $(F_n)_{n < omega}$ be the collection given by the assumption, and let $phi_n$ be the $mathcal{L}$-formula that defines $F_n$, for each $n < omega$. Then for any $N < omega$, by the Finite Intersection Property, we can find a realization $[a_N] in bigcap_{n = 0}^{N} F_n$ such that



      $$mathcal{M} models bigwedge_{n=0}^{N} phi_n ([a_N]).$$



      By Łoś, we have



      $$ left{ i in I : mathcal{M}_i models bigwedge_{n = 0}^{N} phi_n(a_N(i)) right} in mathcal{U}. $$



      That is where I am stuck at. I reckon that my goal is to perhaps show that



      $$left{ i in I : mathcal{M}_i models bigwedge_{n < omega} phi_n(a'(i)) right} in mathcal{U}$$



      and complete the proof by Łoś, but I am not entirely certain if that is what I should be pursuing.










      share|cite|improve this question









      $endgroup$




      I came across the following statement:




      Given an $omega$-sequence of $mathcal{L}$-structures, $(mathcal{M}_i : i < omega)$, and a non-principal ultrafilter $mathcal{U}$ on $omega$, the ultraproduct of the $mathcal{L}$-structures,



      $$ mathcal{M} = prod_{mathcal{U}} M_i $$
      $aleph_1$-compact, i.e. for any countable (non-empty) collection of non-empty definable sets in $mathcal{M}$ that has the Finite Intersection Property, the collection has a non-empty intersection.




      Perhaps the given statement was incomplete (it was in the middle of a lecture, as an aside), but in its proof, it is assumed that the collection is nested, and the rest of the proof inherently relies on this property.



      I wonder if the "nestedness" of the collection is actually a necessary condition. I set out for a proof, but I am unable to arrive at a conclusion. Here is an outline of my attempt:



      Let $(F_n)_{n < omega}$ be the collection given by the assumption, and let $phi_n$ be the $mathcal{L}$-formula that defines $F_n$, for each $n < omega$. Then for any $N < omega$, by the Finite Intersection Property, we can find a realization $[a_N] in bigcap_{n = 0}^{N} F_n$ such that



      $$mathcal{M} models bigwedge_{n=0}^{N} phi_n ([a_N]).$$



      By Łoś, we have



      $$ left{ i in I : mathcal{M}_i models bigwedge_{n = 0}^{N} phi_n(a_N(i)) right} in mathcal{U}. $$



      That is where I am stuck at. I reckon that my goal is to perhaps show that



      $$left{ i in I : mathcal{M}_i models bigwedge_{n < omega} phi_n(a'(i)) right} in mathcal{U}$$



      and complete the proof by Łoś, but I am not entirely certain if that is what I should be pursuing.







      model-theory






      share|cite|improve this question













      share|cite|improve this question











      share|cite|improve this question




      share|cite|improve this question










      asked Dec 5 '18 at 20:33









      JaporizedJaporized

      132




      132






















          1 Answer
          1






          active

          oldest

          votes


















          3












          $begingroup$

          It's not a necessary assumption. If $(D_i)_{iinomega}$ is a family of sets with the finite intersection property, then let $E_i=bigcap_{jle i}D_i$; we have $E_0supseteq E_1supseteq ...$ and each $E_i$ is nonempty by the finite intserction property of the $D_i$s. Moreover, if the $D_i$s were definable, so are the $E_i$s, and anything in the intersection of the $E_i$s is in the intersection of the $D_i$s. So we can go from an arbitrary collection to a nested collection, find something in the intersection of the elements of the nested collection, and say that it's also in the intersection of the elements of the original collection.






          share|cite|improve this answer









          $endgroup$













            Your Answer





            StackExchange.ifUsing("editor", function () {
            return StackExchange.using("mathjaxEditing", function () {
            StackExchange.MarkdownEditor.creationCallbacks.add(function (editor, postfix) {
            StackExchange.mathjaxEditing.prepareWmdForMathJax(editor, postfix, [["$", "$"], ["\\(","\\)"]]);
            });
            });
            }, "mathjax-editing");

            StackExchange.ready(function() {
            var channelOptions = {
            tags: "".split(" "),
            id: "69"
            };
            initTagRenderer("".split(" "), "".split(" "), channelOptions);

            StackExchange.using("externalEditor", function() {
            // Have to fire editor after snippets, if snippets enabled
            if (StackExchange.settings.snippets.snippetsEnabled) {
            StackExchange.using("snippets", function() {
            createEditor();
            });
            }
            else {
            createEditor();
            }
            });

            function createEditor() {
            StackExchange.prepareEditor({
            heartbeatType: 'answer',
            autoActivateHeartbeat: false,
            convertImagesToLinks: true,
            noModals: true,
            showLowRepImageUploadWarning: true,
            reputationToPostImages: 10,
            bindNavPrevention: true,
            postfix: "",
            imageUploader: {
            brandingHtml: "Powered by u003ca class="icon-imgur-white" href="https://imgur.com/"u003eu003c/au003e",
            contentPolicyHtml: "User contributions licensed under u003ca href="https://creativecommons.org/licenses/by-sa/3.0/"u003ecc by-sa 3.0 with attribution requiredu003c/au003e u003ca href="https://stackoverflow.com/legal/content-policy"u003e(content policy)u003c/au003e",
            allowUrls: true
            },
            noCode: true, onDemand: true,
            discardSelector: ".discard-answer"
            ,immediatelyShowMarkdownHelp:true
            });


            }
            });














            draft saved

            draft discarded


















            StackExchange.ready(
            function () {
            StackExchange.openid.initPostLogin('.new-post-login', 'https%3a%2f%2fmath.stackexchange.com%2fquestions%2f3027612%2fultraproducts-with-a-non-principal-ultrafilter-is-aleph-one-compact%23new-answer', 'question_page');
            }
            );

            Post as a guest















            Required, but never shown

























            1 Answer
            1






            active

            oldest

            votes








            1 Answer
            1






            active

            oldest

            votes









            active

            oldest

            votes






            active

            oldest

            votes









            3












            $begingroup$

            It's not a necessary assumption. If $(D_i)_{iinomega}$ is a family of sets with the finite intersection property, then let $E_i=bigcap_{jle i}D_i$; we have $E_0supseteq E_1supseteq ...$ and each $E_i$ is nonempty by the finite intserction property of the $D_i$s. Moreover, if the $D_i$s were definable, so are the $E_i$s, and anything in the intersection of the $E_i$s is in the intersection of the $D_i$s. So we can go from an arbitrary collection to a nested collection, find something in the intersection of the elements of the nested collection, and say that it's also in the intersection of the elements of the original collection.






            share|cite|improve this answer









            $endgroup$


















              3












              $begingroup$

              It's not a necessary assumption. If $(D_i)_{iinomega}$ is a family of sets with the finite intersection property, then let $E_i=bigcap_{jle i}D_i$; we have $E_0supseteq E_1supseteq ...$ and each $E_i$ is nonempty by the finite intserction property of the $D_i$s. Moreover, if the $D_i$s were definable, so are the $E_i$s, and anything in the intersection of the $E_i$s is in the intersection of the $D_i$s. So we can go from an arbitrary collection to a nested collection, find something in the intersection of the elements of the nested collection, and say that it's also in the intersection of the elements of the original collection.






              share|cite|improve this answer









              $endgroup$
















                3












                3








                3





                $begingroup$

                It's not a necessary assumption. If $(D_i)_{iinomega}$ is a family of sets with the finite intersection property, then let $E_i=bigcap_{jle i}D_i$; we have $E_0supseteq E_1supseteq ...$ and each $E_i$ is nonempty by the finite intserction property of the $D_i$s. Moreover, if the $D_i$s were definable, so are the $E_i$s, and anything in the intersection of the $E_i$s is in the intersection of the $D_i$s. So we can go from an arbitrary collection to a nested collection, find something in the intersection of the elements of the nested collection, and say that it's also in the intersection of the elements of the original collection.






                share|cite|improve this answer









                $endgroup$



                It's not a necessary assumption. If $(D_i)_{iinomega}$ is a family of sets with the finite intersection property, then let $E_i=bigcap_{jle i}D_i$; we have $E_0supseteq E_1supseteq ...$ and each $E_i$ is nonempty by the finite intserction property of the $D_i$s. Moreover, if the $D_i$s were definable, so are the $E_i$s, and anything in the intersection of the $E_i$s is in the intersection of the $D_i$s. So we can go from an arbitrary collection to a nested collection, find something in the intersection of the elements of the nested collection, and say that it's also in the intersection of the elements of the original collection.







                share|cite|improve this answer












                share|cite|improve this answer



                share|cite|improve this answer










                answered Dec 5 '18 at 20:46









                Noah SchweberNoah Schweber

                123k10150285




                123k10150285






























                    draft saved

                    draft discarded




















































                    Thanks for contributing an answer to Mathematics Stack Exchange!


                    • Please be sure to answer the question. Provide details and share your research!

                    But avoid



                    • Asking for help, clarification, or responding to other answers.

                    • Making statements based on opinion; back them up with references or personal experience.


                    Use MathJax to format equations. MathJax reference.


                    To learn more, see our tips on writing great answers.




                    draft saved


                    draft discarded














                    StackExchange.ready(
                    function () {
                    StackExchange.openid.initPostLogin('.new-post-login', 'https%3a%2f%2fmath.stackexchange.com%2fquestions%2f3027612%2fultraproducts-with-a-non-principal-ultrafilter-is-aleph-one-compact%23new-answer', 'question_page');
                    }
                    );

                    Post as a guest















                    Required, but never shown





















































                    Required, but never shown














                    Required, but never shown












                    Required, but never shown







                    Required, but never shown

































                    Required, but never shown














                    Required, but never shown












                    Required, but never shown







                    Required, but never shown







                    Popular posts from this blog

                    How do I know what Microsoft account the skydrive app is syncing to?

                    When does type information flow backwards in C++?

                    Grease: Live!